Happy Thanksgiving! Please note that there are no classes November 25th-December 1st.

Contests & Programs AMC and other contests, summer programs, etc.
AMC and other contests, summer programs, etc.
3 M G
BBookmark  VNew Topic kLocked
Contests & Programs AMC and other contests, summer programs, etc.
AMC and other contests, summer programs, etc.
3 M G
BBookmark  VNew Topic kLocked
G
Topic
First Poster
Last Poster
jmo cutoff
cakeiswaybetterthancookie   37
N 16 minutes ago by studymoremath
Source: me
You guys NEED to stop freaking out about JMO cutoffs Its likely going to be in the early-mid 210's assuming an average level aime( 2021 aime?) So if you have a 110+ on the amc 10, you are in contention for USAJMO- its not LIKELY but its possible. My guess is maybe 15ish percent? really, too many people are freaking out about cutoffs- at the end of the day, most of you have next year and the year after that and can always keep improving. You got this!
37 replies
cakeiswaybetterthancookie
Today at 1:10 AM
studymoremath
16 minutes ago
9 When To Start Grinding Math Again
Existing_Human1   142
N 17 minutes ago by MathNerdRabbit103
For context, I got a 103.5 on AMC 10B, so I am not sure if I made AIME. I have AMC 8 and Mathcoutns coming up as competitions
142 replies
Existing_Human1
Nov 23, 2024
MathNerdRabbit103
17 minutes ago
how to prep for AIME
CharviA   4
N 25 minutes ago by orangebear
I'm in 8th grade and I got a 129 on the 10b. For the record I got rly lucky because I guessed e for the last two questions since there were practically no e's on the sheet and there was a lot of combinatorics, which ,fortunately, im relatively good at. This is my first time qualifying for aime and I am currently mocking 3-5. How should I prepare for aime to get at least an 8? thanks
4 replies
CharviA
an hour ago
orangebear
25 minutes ago
No more topics!
generic whiteboard problem
ostriches88   19
N Nov 16, 2024 by happypi31415
Source: 2024 AMC10B p16
Jerry likes to play with numbers. One day, he wrote all the integers from $1$ to $2024$ on the whiteboard. Then he repeatedly chose four numbers on the whiteboard, erased them, and replaced them with either their sum or their product. (For example, Jerry's first step might have been to erase $1, 2, 3$, and $5$, and then write either $11$, their sum, or $30$, their product, on the whiteboard.) After repeatedly performing this operation, Jerry noticed that all the remaining numbers on the board were odd. What is the maximum possible number of integers on the board at that time?

$
\textbf{(A) }1010 \qquad
\textbf{(B) }1011 \qquad
\textbf{(C) }1012 \qquad
\textbf{(D) }1013 \qquad
\textbf{(E) }1014 \qquad
$
19 replies
ostriches88
Nov 13, 2024
happypi31415
Nov 16, 2024
generic whiteboard problem
G H J
G H BBookmark kLocked kLocked NReply
Source: 2024 AMC10B p16
The post below has been deleted. Click to close.
This post has been deleted. Click here to see post.
ostriches88
1454 posts
#1
Y by
Jerry likes to play with numbers. One day, he wrote all the integers from $1$ to $2024$ on the whiteboard. Then he repeatedly chose four numbers on the whiteboard, erased them, and replaced them with either their sum or their product. (For example, Jerry's first step might have been to erase $1, 2, 3$, and $5$, and then write either $11$, their sum, or $30$, their product, on the whiteboard.) After repeatedly performing this operation, Jerry noticed that all the remaining numbers on the board were odd. What is the maximum possible number of integers on the board at that time?

$
\textbf{(A) }1010 \qquad
\textbf{(B) }1011 \qquad
\textbf{(C) }1012 \qquad
\textbf{(D) }1013 \qquad
\textbf{(E) }1014 \qquad
$
This post has been edited 1 time. Last edited by ostriches88, Nov 13, 2024, 6:08 PM
Z K Y
The post below has been deleted. Click to close.
This post has been deleted. Click here to see post.
saturnrocket
1305 posts
#2
Y by
i think the answer's 1011?
Z K Y
The post below has been deleted. Click to close.
This post has been deleted. Click here to see post.
mathMagicOPS
809 posts
#3
Y by
Did anyone else get 1010? I just analyzed each operation (ex. eeee->e)
Z K Y
The post below has been deleted. Click to close.
This post has been deleted. Click here to see post.
ostriches88
1454 posts
#4
Y by
its 1010, you must have at least one of the moves that removes two odd numbers from the board
Z K Y
The post below has been deleted. Click to close.
This post has been deleted. Click here to see post.
eg4334
413 posts
#5
Y by
Note that the total number is invariant mod 3 and the number of odds is nonincreasing. Therefore 1010.
Z K Y
The post below has been deleted. Click to close.
This post has been deleted. Click here to see post.
pingpongmerrily
2543 posts
#6
Y by
this problem fr so generic
can confirm 1010 even tho i didn't solve it
Z K Y
The post below has been deleted. Click to close.
This post has been deleted. Click here to see post.
aleyang
185 posts
#7
Y by
I did OEEE and 1 OOOE
Z K Y
The post below has been deleted. Click to close.
This post has been deleted. Click here to see post.
EaZ_Shadow
378 posts
#8
Y by
BROOOO I CHOKED I GOT C :( it was bc that you can only lose or no gain, so I thought 1012 was the answer, but it wasn't achievable
Z K Y
The post below has been deleted. Click to close.
This post has been deleted. Click here to see post.
lucaswujc
290 posts
#9
Y by
saturnrocket wrote:
i think the answer's 1011?

was 1010 iirc
Z K Y
The post below has been deleted. Click to close.
This post has been deleted. Click here to see post.
megarnie
5324 posts
#10
Y by
The answer is $\boxed{\textbf{(A)}\ 1010}$.

Bound: Note that each move removes at most $3$ evens, and there are $1012$ evens, meaning we must make at least $338$ operations and therefore lose at least $338 \cdot 3 = 1014$ numbers.

Construction (didn't need this in test):

Claim: You can turn $4n$ evens into $n$ evens in $n$ turns.
Proof: Just group them into $n$ groups of $4$ and operate on each one. $\square$

Extending this claim, we can also turn $4n + k$ evens into $n + k$ evens in $n$ operations.

Note that the number of evens is initially $1012$. Consider $1012 \to 253 \to 64 \to 16 \to 4 \to 1$ takes $253 + 63 + 16 + 4 + 1 = 337$ operations. Now operate the final even with three random odds and we are done.
This post has been edited 1 time. Last edited by megarnie, Nov 13, 2024, 6:19 PM
Z K Y
The post below has been deleted. Click to close.
This post has been deleted. Click here to see post.
andrewcheng
482 posts
#11
Y by
notice that each operation takes 3 numbers out
to remove all evens we must remove at least 1012 entries(all the evens)
1014 is the smallest mult of 3 >1012
2024-1014=1010
Z K Y
The post below has been deleted. Click to close.
This post has been deleted. Click here to see post.
alexanderhamilton124
250 posts
#12
Y by
I got A; did (O, O, O, E) operations, followed by one last (E, O, O, O).
Z K Y
The post below has been deleted. Click to close.
This post has been deleted. Click here to see post.
elizhang101412
1068 posts
#13
Y by
i tried to cheese this by finding 2024-3x; should have went with 1010

now that I think about it 1013 isn't even realistically possible idk what I was on :(
Z K Y
The post below has been deleted. Click to close.
This post has been deleted. Click here to see post.
giratina3
247 posts
#14
Y by
I got 1009, but realized that I didn’t got the new odd numbers formed and got 1010
Z K Y
The post below has been deleted. Click to close.
This post has been deleted. Click here to see post.
KevinChen_Yay
136 posts
#15
Y by
I got 1010. Every time the total amount of integers decreases by 3 because it's 4-->1, therefor the final answer is either (A) or (D). However, since the amount of odd integers can only decrease (the only case where it may increase is all 4 even --> 1 odd but that's impossible), the answer is $\boxed{\mathbf{(A)}\ 1010}$.
Z K Y
The post below has been deleted. Click to close.
This post has been deleted. Click here to see post.
joshualiu315
2431 posts
#16
Y by
the answer choices here are so stupid bruh


Note that each operation removes three numbers from the whiteboard. It is easy to see that there are many ways to have all three removed numbers be even for each move, but we still need at least $338$ operations to remove $1012$ even numbers. Hence, the answer is $\boxed{1010}$ (some odd numbers will be removed on the last operation).


Remark: the only possible answer choices are A and D lol
This post has been edited 1 time. Last edited by joshualiu315, Nov 13, 2024, 8:51 PM
Z K Y
The post below has been deleted. Click to close.
This post has been deleted. Click here to see post.
andrewcheng
482 posts
#17
Y by
I threw this by getting that multiplying 4 es only subtracted 2 I had doubts about my ans here
Z K Y
The post below has been deleted. Click to close.
This post has been deleted. Click here to see post.
FuturePanda
143 posts
#18
Y by
Yes! Me got it right!
Z K Y
The post below has been deleted. Click to close.
This post has been deleted. Click here to see post.
Tetra_scheme
22 posts
#19
Y by
during the test this problem gave me hope the 10b was high quality. The total is invariant mod 3 and there can't be more odds so it is the next number that is equal to $2$ mod 3 or 1010.
Z K Y
The post below has been deleted. Click to close.
This post has been deleted. Click here to see post.
happypi31415
668 posts
#20
Y by
just note that it must be less then or equal to $1012$, because there are $1012$ odd numbers, and the answer is invariant $\pmod {3}$, which gives the only possible answer as $\boxed{\textbf{(A)}\ 1010}$
Z K Y
N Quick Reply
G
H
=
a